Difference between revisions of "2015 AMC 12B Problems/Problem 5"

(Created page with "==Problem== ==Solution== ==See Also== {{AMC12 box|year=2015|ab=B|num-a=6|num-b=4}} {{MAA Notice}}")
 
(Problem)
Line 1: Line 1:
 
==Problem==
 
==Problem==
 +
The Tigers beat the Sharks 2 out of the 3 times they played. They then played <math>N</math> more times, and the Sharks ended up winning at least 95% of all the games played. What is the minimum possible value for <math>N</math>?
  
 
+
<math>\textbf{(A)}\; ? \qquad\textbf{(B)}\; ? \qquad\textbf{(C)}\; ? \qquad\textbf{(D)}\; ? \qquad\textbf{(E)}\; ?</math>
  
 
==Solution==
 
==Solution==

Revision as of 14:18, 3 March 2015

Problem

The Tigers beat the Sharks 2 out of the 3 times they played. They then played $N$ more times, and the Sharks ended up winning at least 95% of all the games played. What is the minimum possible value for $N$?

$\textbf{(A)}\; ? \qquad\textbf{(B)}\; ? \qquad\textbf{(C)}\; ? \qquad\textbf{(D)}\; ? \qquad\textbf{(E)}\; ?$

Solution

See Also

2015 AMC 12B (ProblemsAnswer KeyResources)
Preceded by
Problem 4
Followed by
Problem 6
1 2 3 4 5 6 7 8 9 10 11 12 13 14 15 16 17 18 19 20 21 22 23 24 25
All AMC 12 Problems and Solutions

The problems on this page are copyrighted by the Mathematical Association of America's American Mathematics Competitions. AMC logo.png